2020 AMC 10A Problems/Problem 11

Revision as of 21:49, 31 January 2020 by Pcchess (talk | contribs) (Created page with "{{duplicate|2020 AMC 12A #8 and 2020 AMC 10A #11}} ==Problem 11== What is the median of the following list of <math>4040<...")
(diff) ← Older revision | Latest revision (diff) | Newer revision → (diff)
The following problem is from both the 2020 AMC 12A #8 and 2020 AMC 10A #11, so both problems redirect to this page.

Problem 11

What is the median of the following list of $4040$ numbers$?$

\[1, 2, 3, ..., 2020, 1^2, 2^2, 3^2, ..., 2020^2\]

$\textbf{(A)}\ 1974.5\qquad\textbf{(B)}\ 1975.5\qquad\textbf{(C)}\ 1976.5\qquad\textbf{(D)}\ 1977.5\qquad\textbf{(E)}\ 1978.5$

==Solution